Last visit was: 25 Apr 2024, 13:24 It is currently 25 Apr 2024, 13:24

Close
GMAT Club Daily Prep
Thank you for using the timer - this advanced tool can estimate your performance and suggest more practice questions. We have subscribed you to Daily Prep Questions via email.

Customized
for You

we will pick new questions that match your level based on your Timer History

Track
Your Progress

every week, we’ll send you an estimated GMAT score based on your performance

Practice
Pays

we will pick new questions that match your level based on your Timer History
Not interested in getting valuable practice questions and articles delivered to your email? No problem, unsubscribe here.
Close
Request Expert Reply
Confirm Cancel
SORT BY:
Date
Tags:
Difficulty: 505-555 Levelx   Percent and Interest Problemsx                     
Show Tags
Hide Tags
User avatar
Manager
Manager
Joined: 02 Dec 2012
Posts: 172
Own Kudos [?]: 23855 [108]
Given Kudos: 23
Send PM
Most Helpful Reply
Math Expert
Joined: 02 Sep 2009
Posts: 92915
Own Kudos [?]: 619010 [21]
Given Kudos: 81595
Send PM
User avatar
Manager
Manager
Joined: 10 May 2014
Posts: 116
Own Kudos [?]: 339 [11]
Given Kudos: 28
Send PM
General Discussion
avatar
Intern
Intern
Joined: 22 Aug 2013
Posts: 2
Own Kudos [?]: [0]
Given Kudos: 0
Send PM
Re: The price per share of stock X increased by 10 percent over [#permalink]
Sorry All but I was not able to view the options given. I am using Firefox browser.
Math Expert
Joined: 02 Sep 2009
Posts: 92915
Own Kudos [?]: 619010 [0]
Given Kudos: 81595
Send PM
Re: The price per share of stock X increased by 10 percent over [#permalink]
Expert Reply
SrirFwd wrote:
Sorry All but I was not able to view the options given. I am using Firefox browser.


This is a data sufficiency question. Options for DS questions are always the same.

The data sufficiency problem consists of a question and two statements, labeled (1) and (2), in which certain data are given. You have to decide whether the data given in the statements are sufficient for answering the question. Using the data given in the statements, plus your knowledge of mathematics and everyday facts (such as the number of days in July or the meaning of the word counterclockwise), you must indicate whether—

A. Statement (1) ALONE is sufficient, but statement (2) alone is not sufficient to answer the question asked.
B. Statement (2) ALONE is sufficient, but statement (1) alone is not sufficient to answer the question asked.
C. BOTH statements (1) and (2) TOGETHER are sufficient to answer the question asked, but NEITHER statement ALONE is sufficient to answer the question asked.
D. EACH statement ALONE is sufficient to answer the question asked.
E. Statements (1) and (2) TOGETHER are NOT sufficient to answer the question asked, and additional data specific to the problem are needed.

Hope this helps.
Intern
Intern
Joined: 02 Jan 2017
Posts: 47
Own Kudos [?]: 15 [0]
Given Kudos: 23
Location: Pakistan
Concentration: Finance, Technology
GMAT 1: 650 Q47 V34
GPA: 3.41
WE:Business Development (Accounting)
Send PM
Re: The price per share of Stock X increased by 10 percent over [#permalink]
Bunuel wrote:
The price per share of Stock X increased by 10 percent over the same time period that the price per share of Stock Y decreased by 10 percent. The reduced price per share of Stock Y was what percent of the original price per share of Stock X ?

Let the initial price per share of stock X be \(x\), so after increase by 10% it would become \(1.1x\);
Let the initial price per share of stock Y be \(y\), so after decrease by 10% it would become \(0.9y\).

Question: \(\frac{0.9y}{x}=\frac{9y}{10x}=?\)

(1) The increased price per share of Stock X was equal to the original price per share of Stock Y --> \(1.1x=y\) --> \(\frac{9y}{10x}=\frac{9*1.1x}{10x}=0.99\) or 99%. Sufficient.

(2) The increase in the price per share of Stock X was 10/11 the decrease in the price per share of Stock Y --> \(1.1x-x=\frac{10}{11}*(y-0.9y)\) --> \(0.1x=\frac{10}{11}*0.1y\) --> \(1.1x=y\) the same info as in (1). Sufficient

Answer: D.



Shouldn't the highlighted part be [ y- .1y] as its 10/11 of the decease and it is to be decreased by 10 %, correct me if I'm wrong.
Regards
Attachments

g club.PNG
g club.PNG [ 13.86 KiB | Viewed 25403 times ]

Math Expert
Joined: 02 Sep 2009
Posts: 92915
Own Kudos [?]: 619010 [0]
Given Kudos: 81595
Send PM
Re: The price per share of Stock X increased by 10 percent over [#permalink]
Expert Reply
mtk10 wrote:
Bunuel wrote:
The price per share of Stock X increased by 10 percent over the same time period that the price per share of Stock Y decreased by 10 percent. The reduced price per share of Stock Y was what percent of the original price per share of Stock X ?

Let the initial price per share of stock X be \(x\), so after increase by 10% it would become \(1.1x\);
Let the initial price per share of stock Y be \(y\), so after decrease by 10% it would become \(0.9y\).

Question: \(\frac{0.9y}{x}=\frac{9y}{10x}=?\)

(1) The increased price per share of Stock X was equal to the original price per share of Stock Y --> \(1.1x=y\) --> \(\frac{9y}{10x}=\frac{9*1.1x}{10x}=0.99\) or 99%. Sufficient.

(2) The increase in the price per share of Stock X was 10/11 the decrease in the price per share of Stock Y --> \(1.1x-x=\frac{10}{11}*(y-0.9y)\) --> \(0.1x=\frac{10}{11}*0.1y\) --> \(1.1x=y\) the same info as in (1). Sufficient

Answer: D.



Shouldn't the highlighted part be [ y- .1y] as its 10/11 of the decease and it is to be decreased by 10 %, correct me if I'm wrong.
Regards


It's should be the way it is written.

The decrease in the price per share of Stock Y is \(y - 0.9y\). 10/11 times that is \(\frac{10}{11}*(y-0.9y)\). I think you misunderstood the statement.
Intern
Intern
Joined: 02 Jan 2017
Posts: 47
Own Kudos [?]: 15 [0]
Given Kudos: 23
Location: Pakistan
Concentration: Finance, Technology
GMAT 1: 650 Q47 V34
GPA: 3.41
WE:Business Development (Accounting)
Send PM
Re: The price per share of Stock X increased by 10 percent over [#permalink]
Bunuel wrote:
mtk10 wrote:
Bunuel wrote:
The price per share of Stock X increased by 10 percent over the same time period that the price per share of Stock Y decreased by 10 percent. The reduced price per share of Stock Y was what percent of the original price per share of Stock X ?

Let the initial price per share of stock X be \(x\), so after increase by 10% it would become \(1.1x\);
Let the initial price per share of stock Y be \(y\), so after decrease by 10% it would become \(0.9y\).

Question: \(\frac{0.9y}{x}=\frac{9y}{10x}=?\)

(1) The increased price per share of Stock X was equal to the original price per share of Stock Y --> \(1.1x=y\) --> \(\frac{9y}{10x}=\frac{9*1.1x}{10x}=0.99\) or 99%. Sufficient.

(2) The increase in the price per share of Stock X was 10/11 the decrease in the price per share of Stock Y --> \(1.1x-x=\frac{10}{11}*(y-0.9y)\) --> \(0.1x=\frac{10}{11}*0.1y\) --> \(1.1x=y\) the same info as in (1). Sufficient

Answer: D.





Shouldn't the highlighted part be [ y- .1y] as its 10/11 of the decease and it is to be decreased by 10 %, correct me if I'm wrong.
Regards


It's should be the way it is written.

The decrease in the price per share of Stock Y is \(y - 0.9y\). 10/11 times that is \(\frac{10}{11}*(y-0.9y)\). I think you misunderstood the statement.





But the stem states that decrease in y is 10 %, Would we not write this question 1 -.1 =.9 . Sorry, i guess I am overlooking something. I am not understanding why are you subtracting .9y when question states that decrease is 10 % not 90 %.
Kindly explain
Math Expert
Joined: 02 Sep 2009
Posts: 92915
Own Kudos [?]: 619010 [1]
Given Kudos: 81595
Send PM
Re: The price per share of Stock X increased by 10 percent over [#permalink]
1
Bookmarks
Expert Reply
mtk10 wrote:
But the stem states that decrease in y is 10 %, Would we not write this question 1 -.1 =.9 . Sorry, i guess I am overlooking something. I am not understanding why are you subtracting .9y when question states that decrease is 10 % not 90 %.
Kindly explain



The price per share of Stock Y decreased by 10 percent:
Let the initial price per share of stock Y be \(y\), so after decrease by 10% it would become \(0.9y\).
Amount by which it decreased is initial price - new price = y-0.9y.
Tutor
Joined: 12 Oct 2010
Status:GMATH founder
Posts: 893
Own Kudos [?]: 1355 [2]
Given Kudos: 56
Send PM
Re: The price per share of Stock X increased by 10 percent over [#permalink]
2
Kudos
Expert Reply
Walkabout wrote:
The price per share of Stock X increased by 10 percent over the same time period that the price per share of Stock Y decreased by 10 percent. The reduced price per share of Stock Y was what percent of the original price per share of Stock X ?

(1) The increased price per share of Stock X was equal to the original price per share of Stock Y.
(2) The increase in the price per share of Stock X was 10/11 the decrease in the price per share of Stock Y.


Important: Bunuel´s explanations (above) ARE FUNDAMENTAL to students who are beginning their "GMAT Quant development".
The solution we present below is for the students who already know "the basics".

(That´s why we postpone Data Sufficiency learning, in our course, till the student´s maturity has changed considerably!)

In our method we make a clear distinction between Problem Solving and Data Sufficiency.

In Data Sufficiency problems, we are FOCUSED in the UNIQUENESS OR NOT of a (numerical) value, not in calculations related to the possible value(s)...

Please read our solution, that starts below, with that in mind!

\(\left( {{\text{DATA}}} \right)\,\,\,\,X\,\,\, \to \,\,\,\frac{{11}}{{10}}X\,\,\,\,\,\,\,\,;\,\,\,\,\,\,\,\,Y\,\,\, \to \,\,\,\frac{9}{{10}}Y\)

\(\left( {{\text{FOCUS}}} \right)\,\,\,\,? = \frac{{\,\frac{9}{{10}}Y\,}}{X}\,\,\,\,\,\, \Leftrightarrow \,\,\,\,\,\boxed{? = \frac{Y}{X}}\,\,\,\,\,\,\,\,\,\,\,\left( {X \ne 0\,\,\,\,{\text{implicitly}}} \right)\)

\(\left( 1 \right)\,\,\,\frac{{11}}{{10}}X = Y\,\,\,\, \Rightarrow \,\,\,\,\,\frac{Y}{X}\,\,\,{\text{unique}}\,\,\,\, \Rightarrow \,\,\,\,\,{\text{SUFF}}.\)

\(\left( 2 \right)\,\,\,\frac{1}{{10}}X = \frac{{10}}{{11}}\left( {\frac{1}{{10}}Y} \right)\,\,\,\, \Rightarrow \,\,\,\,\,\frac{Y}{X}\,\,\,{\text{unique}}\,\,\,\, \Rightarrow \,\,\,\,\,{\text{SUFF}}.\)

(If you realize this is BRUTAL, SAFE and POWERFUL, you understood correctly. )


This solution follows the notations and rationale taught in the GMATH method.

Regards,
Fabio.
Manager
Manager
Joined: 10 Apr 2018
Posts: 187
Own Kudos [?]: 448 [0]
Given Kudos: 115
Location: United States (NC)
Send PM
Re: The price per share of Stock X increased by 10 percent over [#permalink]
Here are my two cents for this question:

Attachment:
Sample.png
Sample.png [ 6.45 KiB | Viewed 18985 times ]


We are asked .9b is what % of a ----- (I)

From statement 1 we have:

1.1 a= b
or
a=\(\frac{10}{11} b\)

Since we have a relation between a and b we can definitely solve for the equation (I)
.9b=\(\frac{x}{100} \frac{10}{11} b\)

Statement 2 we have:
Increase in per share price of stock X is .1 a , and\( \frac{10}{11}\) decrease in price per share of stock y is \(\frac{1}{11} b \)
and we are told that .1a = \(\frac{1}{11} b \)
or a=\( \frac{10}{11}\) b which is essentially same as statement 1 .

So both 1 and 2 Statement individually help to answer our question
Re: The price per share of Stock X increased by 10 percent over [#permalink]
mtk10 wrote:
Bunuel wrote:
The price per share of Stock X increased by 10 percent over the same time period that the price per share of Stock Y decreased by 10 percent. The reduced price per share of Stock Y was what percent of the original price per share of Stock X ?

Let the initial price per share of stock X be \(x\), so after increase by 10% it would become \(1.1x\);
Let the initial price per share of stock Y be \(y\), so after decrease by 10% it would become \(0.9y\).

Question: \(\frac{0.9y}{x}=\frac{9y}{10x}=?\)

(1) The increased price per share of Stock X was equal to the original price per share of Stock Y --> \(1.1x=y\) --> \(\frac{9y}{10x}=\frac{9*1.1x}{10x}=0.99\) or 99%. Sufficient.

(2) The increase in the price per share of Stock X was 10/11 the decrease in the price per share of Stock Y --> \(1.1x-x=\frac{10}{11}*(y-0.9y)\) --> \(0.1x=\frac{10}{11}*0.1y\) --> \(1.1x=y\) the same info as in (1). Sufficient

Answer: D.



But the stem states that decrease in y is 10 %, Would we not write this question 1 -.1 =.9 . Sorry, i guess I am overlooking something. I am not understanding why are you subtracting .9y when question states that decrease is 10 % not 90 %.
Kindly explain


mtk10
the \(decrease\) in the price per share of Stock Y is \(0.1y\). (The decreased price is 0.9y). Both are not the same cases.
Current Student
Joined: 26 May 2019
Posts: 737
Own Kudos [?]: 263 [0]
Given Kudos: 84
Location: India
GMAT 1: 650 Q46 V34
GMAT 2: 720 Q49 V40
GPA: 2.58
WE:Consulting (Consulting)
Send PM
The price per share of Stock X increased by 10 percent over [#permalink]
Please correct if i'm wrong here. Assuming original prices are X and Y, we need to find 0.9Y/X*100.

St. 1 -- Sufficient. It tells us 1.1X = Y. We can find out 0.9Y/X %.
St. 2 -- Sufficient. It tells us 0.1X = (10/11)*0.1Y. We can again find out 0.9Y/X % through the given information.

Therefore, D is the correct answer.
User avatar
Non-Human User
Joined: 09 Sep 2013
Posts: 32679
Own Kudos [?]: 822 [0]
Given Kudos: 0
Send PM
Re: The price per share of Stock X increased by 10 percent over [#permalink]
Hello from the GMAT Club BumpBot!

Thanks to another GMAT Club member, I have just discovered this valuable topic, yet it had no discussion for over a year. I am now bumping it up - doing my job. I think you may find it valuable (esp those replies with Kudos).

Want to see all other topics I dig out? Follow me (click follow button on profile). You will receive a summary of all topics I bump in your profile area as well as via email.
GMAT Club Bot
Re: The price per share of Stock X increased by 10 percent over [#permalink]
Moderator:
Math Expert
92914 posts

Powered by phpBB © phpBB Group | Emoji artwork provided by EmojiOne